Ch 40- Terrorism Response and Disaster Management

Your page rank:

Total word count: 15933
Pages: 58

Calculate the Price

- -
275 words
Looking for Expert Opinion?
Let us have a look at your work and suggest how to improve it!
Get a Consultant

Early signs and symptoms of smallpox include all of the following, EXCEPT:

skin blisters

Nerve agents, a class of chemicals called organophosphates, were first discovered while in search of a superior:

pesticide

A 52-year-old male presents with a fever of 102.5°F and a severe headache. As you assess him, you note the presence of multiple blisters on his face and chest, which are all identical in shape and size. This patient’s clinical presentation is MOST consistent with:

smallpox

As the first arriving emergency responder at the scene of a suspected terrorist or WMD incident, you should request additional resources as needed and then:

function as the incident commander until additional personnel arrive

At present, the likelihood of a nuclear attack against the United States is very low because:

terrorist nations do not have the ability to deliver a nuclear weapon via missile or bomb

The skin lesions associated with smallpox:

are identical in there development

Continual reassessment of the scene at a suspected terrorist or WMD incident is MOST important because:

a secondary explosive device may detonate

Which of the following statements regarding the persistency and volatility of a chemical agent is correct?

VX, a highly persistent nerve agent, can remain in the environment for weeks to months.

Initial signs and symptoms associated with viral hemorrhagic fevers include:

headache and sore throat

The primary route of exposure of vesicant agents is the:

skin

Signs and symptoms of exposure to a nerve agent include:

salivation, pinpoint pupils, and diarrhea.

In determining the potential for a terrorist attack, you should routinely observe all of the following on every call, EXCEPT:

weather conditions

Unlike viruses and bacteria, neurotoxins

are not contagious

Which of the following is NOT a factor in determining how to protect oneself against the effects of radiation?

body size

Pulmonary hemorrhage and inner ear damage are examples of __________ blast injuries.

primary

You and your partner arrive at the scene of a fire at a large office complex. Witnesses tell you that they heard a loud explosion shortly before the building caught fire. You should

ensure that your ambulance is parked upwind and uphill from the building

Botulinum is:

a potent bacterial neurotoxin

When multiple patients present with an acute onset of difficulty breathing, chest tightness, and hoarseness or stridor, you should be MOST suspicious of exposure to

phosgene or chlorine

Which of the following statements regarding a "dirty bomb" is correct?

Dirty bombs could injure victims with both radioactive material and the explosive material used to deliver it

Which of the following statements regarding anthrax is correct?

Pulmonary anthrax is the most deadly form

Which of the following chemicals is a nerve agent?

soman

The chemical attacks that occurred in Tokyo between 1994 and 1995 were carried out by a

violent religious group

You are treating a patient who experienced a significant exposure to cyanide. He is semiconscious and is breathing inadequately. The MOST appropriate method of providing assisted ventilations to this patient is to:

use a bag-mask device

The process performed to artificially maximize the target population’s exposure to a biologic agent, thereby exposing the greatest number of people and achieving the desired effect, is called:

weaponization

All of the following are vesicant agents, EXCEPT

sarin

Continual reassessment of the scene at a suspected terrorist or WMD incident is MOST important because

a secondary explosive device may detonate.

A disease vector is defined as:

any agent that acts as a carrier or transporter

The primary clinical feature associated with exposure to phosgene oxime is:

skin blistering

According to the Department of Homeland Security Security Advisory System, the color orange indicates a _______ risk of terrorist attacks.

high

To date, the preferred weapons of mass destruction for terrorists have been

explosive weapons

An attack on an abortion clinic would MOST likely be carried out by a

single-issue group

A weapon of mass destruction (WMD) is MOST accurately defined as:

any agent used to bring about mass death, casualties, or massive infrastructural damage.

Most terrorist attacks:

require multiple terrorists working together

Viral hemorrhagic fevers (VHF), such as Ebola, cause the blood to

seep out of the blood vessels and tissues.

A persistent or nonvolatile chemical agent can:

remain on a surface for more than 24 hours

Points of distribution (PODs) are strategically placed facilities where

antidotes, antibiotics, and vaccines are distributed.

Unlike bacterial agents, viral agents:

are usually not treatable

If the incident command system (ICS) is already established at the scene of a WMD or terrorist attack, the EMT should:

locate the medical staging officer to obtain his or her assignment.

The Centennial Park bombing during the 1996 Summer Olympics is an example of:

domestic terrorism

The type and severity of wounds sustained from incendiary and explosive devices primarily depend on the:

patient’s distance from the epicenter of the explosion.

After eating at a local restaurant, a 20-year-old male complains of blurred vision, difficulty speaking, and difficulty breathing. He is conscious; however, his respirations are profoundly labored and producing minimal tidal volume. You should:

assist his ventilations with 100% oxygen.

Multiple people in a small town began experiencing abdominal cramps, excessive salivation and urination, and muscle twitching shortly after a small crop duster plane made several passes over the community. As you are assessing the patients, you further determine that most of them are bradycardic and have miosis. In addition to 100% oxygen, the MOST important treatment for these patients includes:

atropine and pralidoxime chloride (2-PAM).

All of the following biologic agents or diseases can be transmitted from person to person, EXCEPT:

ricin

Which of the following statements regarding blast injuries is correct?

Solid organs are relatively protected from shock wave injury but may be injured during the secondary or tertiary blast

Exposure to _____ would MOST likely result in immediate respiratory distress?

chlorine

The LEAST harmful form of ionizing radiation is?

Alpha

When introduced into the body, ricin causes?

pulmonary edema and circulatory failure.

The chemical attacks that occurred in Tokyo between 1994 qnd 1995 were carried out by a?

A violent religious group

A severe risk of terrorist attacks is indicated by the color ______ by the Department of Homeland Security Advisory System?

red

international terrorism (also known as cross-border terrorism)

Terrorism that is carried out by people in a country other than their own

1. Violent religious groups/doomsday cults
-include groups such as Aum Shinrikyo (tokyo subway)
-may participate in apocalyptic violence

2. Extremist political groups
-may include violent separatist groups and those who seek political, religious, economic, and social freedom
-e.g. Taliban militant groups

3. Technology terrorists
-those who attack a population’s technological infrastructure as a means to draw attention to their cause, such as cyberterrorists

4. Single-issue groups
-these include antiabortion groups, animal rights groups, anarchists, racists, and even ecoterrorists who threaten or use violence as a means to protect the environment

What are the different types of groups that may turn toward terrorism as a means to achieve their goals?

weapon of mass destruction (WMD) or weapon of mass casualty (WMC)

-these instruments of death and destruction include biologic, nuclear, incendiary, chemical, and explosive weapons (B-NICE), or chemical, biologic, radiologic, nuclear, and explosive (CBRNE) weaponds

-to date, the preferred WMD for terrorists has been explosive devices

-WMDs are relatively easy to obtain or create and are specifically geared toward killing large numbers of people
-the technical recipes for making B-NICE weapons can be found readily online

-chemical warfare may consist of agents in the form of a liquid, powder, or vapor

Any agent designed to bring about mass death, casualties, and/or massive damage to property and infrastructures (bridges, tunnels, airports, and seaports)

Chemical agents are manufactured substances that can have devastating effects on living organisms
-they can be produced in liquid, powder, or vapor form depending on desired route of exposure and dissemination technique
-developed during WWI, used on battlefield and have been used to terrorize civilian populations

Chemical warfare agents include:
-vesicants (blister agents)
-respiratory agents (choking agents)
-nerve agents
-metabolic agents (cyanides)

Chemical Terrorism/Warfare

Biologic agents are organisms that cause disease
-generally found in nature, however for terrorist use they are cultivated, synthesized, and mutated in a lab
-the weaponization of biologic agents is performed to artificially maximize the target population’s exposure to the germ, thereby exposing the greatest number of people and achieving the desired result

Primary types of biologic agents that you may come into contact with during a biologic event include:
-viruses
-bacteria
-toxins

Biologic Terrorism/Warfare

weaponization

The creation of a weapon from a biologic agent generally found in nature and that causes disease; the agent is cultivated, synthesized, and/or mutated to maximize the target population’s exposure to the germ

-includes use of nuclear bombs, such as those used during WWII when Hiroshima and Nagasaki were targeted with nuclear bombs
-there are also nations that hold close ties with terrorist groups (known as state-sponsored terrorism) and have obtained some degree of nuclear capability
-it is also possible for a terrorist to secure radioactive materials or waste to perpetuate an act of terror –> these materials are easier to acquire and require less expertise to use. Radioactive materials (such as those in radiologic dispersal devices (RDDs) also known as "dirty bombs") can cause widespread panic and civil disturbances

Nuclear/Radiologic Terrorism

in large-scale terrorist events, it is important to use triage and base patient care on available resources

EMT Response to Terrorism

-most terrorist attacks are covert, meaning the public safety community generally has no prior knowledge of the time, location, or nature of the attack
-the Homeland Security Advisory System alerts responders to the potential for an attack, although specifics of the current threat will not be given
-on the basis of the current threat level, EMTs should take appropriate actions and precautions while continuing to perform daily duties and responding to calls
-The system of colors is used to inform the public safety community of the climate of terrorism and to heighten the awareness of the potential for a terrorist attack
-the system is designed to save lives
-DHS has not issued specific recommendations for EMS personnel to follow in response to the alert system. Follow local protocols and procedures

-it is your responsibility to make sure you know the advisory level at the start of your workday (DHS website, newspaper, TV, etc. all give up-to-date info on the threat level)

Recognizing a Terrorist Event (Indicators)

1. Type of location
-is the location a monument, infrastructure, government building, or a specific type of location like a temple?
-is there a large gathering or special event taking place?

2. Type of call
-is there a report of an explosion or suspicious device nearby?
-does the call come into dispatch as someone having unexplained coughing and difficulty breathing?
-are there reports of people fleeing the scene?

3. Number of patients
-are there multiple victims with similar signs and symptoms?
^^this is the single most important clue that a terrorist attack or incident involving WMD has occurred

4. Victim’s statements
-second best indication of a terrorist or WMD event
-are victims fleeing the scene giving statements such as, "Everyone is passing out," "there was a loud explosion," or "there are a lot of people shaking on the ground"
-if so, something is occurring that you do not want to rush into, even if it is determined not to be a terrorist event

5. Preincident indicators
-is the terror alert level high (orange) or severe (red)?
-has there been a recent increase in violent political activism?
-are you aware of any credible threats made against the location, gathering, or occasion?

In determining the potential for a terrorist attack, on every call you should make the following observations (5)

multiple victims with similar signs and symptoms

-second best indication of a terrorist or WMD event = victim’s statements

What is the single most important clue that a terrorist attack or an incident involving a WMD has occurred?

Severe (RED)
-severe risk of terrorist attack

High (ORANGE)
-high risk of terrorist attack

Elevated (YELLOW)
-significant risk of terrorist attack

Guarded (BLUE)
-general risk of terrorist attack

Low (GREEN)
-low risk of terrorist attack

The Department of Homeland Security Advisory System is posted daily to heighten awareness of the current terrorist threat. What are the 5 threat levels in this system?

Look at patients’ presenting signs and symptoms

-the intentional use of a WMD affects multiple persons and these casualties will generally exhibit the same signs and symptoms
-it is highly unlikely for more than one person to experience a seizure at any given time

What is one easy way to distinguish between a nonterrorist MCI and a terrorist event?

1. Scene safety
2. Responder Safety (personal protection)
3. Notification procedures
4. Establishing command
5. Secondary device or event (reassessing scene safety)

Response Actions — once you suspect that a terrorist event has occurred or a WMD has been used, there are certain actions you must take to ensure that you will be safe and be in the proper position to help the community (5)

1. Scene Safety
-stage your vehicle a safe distance (1-2 blocks) from the incident and wait for law enforcement to advise you that scene has been made secure
-in a suspected WMD incident, it’s best to stage uphill and upwind of the incident and wait for assistance from those who are trained in assessing and managing WMD scenes
-law enforcement will most likely make a perimeter

Remember following rules:
-always have an escape plan determined beforehand in case scene becomes unsafe
-make sure that your vehicle is not blocked in by other emergency vehicles or damaged by a secondary device because then you will be unable to provide victims with transportation or escape yourself

2. Responder Safety (Personal Protection)
-the greatest threats facing you in a WMD attack are contamination and cross-contamination
-contamination with an agent occurs when you have direct contact with a WMD or are exposed to it
-cross-contamination occurs when you come into contact with a contaminated person who has not yet been decontaminated

3. Notification Procedures
-when you suspect a terrorist or WMD event has taken place, notify dispatcher if communications are functioning properly
-inform dispatch of the nature of event, any additional resources that may be required, the estimated number of patients, and the upwind route of approach or optimal route of approach
-establish a staging area where other units will converge
-trained responders in proper protective equipment are the only persons equipped to handle the WMD incident (HazMat teams typically) and should be requested as early as possible

4. Establishing command
-first arriving provider on the scene must begin to sort out the chaos and define his responsibilities under the ICS
-as the first person on scene, the EMT may need to establish command until additional personnel arrive
-if the initial ICS is already in place when you arrive, immediately seek out the medical staging officer to receive your assignment

5. Secondary Device or Event (Reassessing Scene Safety)
-there may be a secondary device that is set to explode after initial bomb and these secondary devices are intended primarily to injure responders and to secure media coverage
-secondary devices may include various types of electronic equipment such as cell phones or pagers that are detonated when "answered"
-constantly assess and reassess scene for safety
-something as subtle as a change in wind direction during a gas attack or an increase in the number of contaminated patients can place you in danger

Response Actions — once you suspect that a terrorist event has occurred or a WMD has been used, there are certain actions you must take to ensure that you will be safe and be in the proper position to help the community (5 explained)

= liquids or gases that are dispersed to kill or injure

-chemical weapons have several classifications
-the properties or characteristics of an agent can be described as a liquid, gas, or solid material
-persistency and volatility are terms used to describe how long the agent will stay on a surface before it evaporates
-persistent or nonvolatile agents can remain on a surface for long periods (usually longer than 24 hours), whereas nonpersistent or volatile agents evaporate relatively fast when left on a surface in the optimal temperature range
-an agent that is described as highly persistent (e.g. VX, a nerve agent) can remain in the environment for weeks/months, whereas an agent that is highly volatile (e.g. sarin, a nerve agent) will turn from liquid to gas (evaporate) within minutes to seconds

-route of exposure = manner in which toxic substance enters body
-chemical agents can have either a vapor or contact hazard
-Agents with a vapor hazard enter the body through the respiratory tract in the form of vapors
-Agents with a contact hazard (or skin hazard) give off very little vapor or no vapors and enter the body through the skin

Chemical agents

evaporates quickly;
remains on a surface a long time before it evaporates (possibly weeks to months)

If a chemical agent is highly volatile, it _______, whereas if it’s highly persistent, it _____.

VX, a nerve agent

sarin, another nerve agent

What’s an example of a highly persistent chemical agent that can remain in the environment for weeks-months? What’s an example of a highly volatile chemical agent that rapidly evaporates?

1. Vesicants (blister agents)
-sulfur mustard (H)
-Lewisite (L)
-phosgene oxime (CX)

2. Pulmonary Agents (Choking Agents)
-chlorine (CL)
-Phosgene

3. Nerve Agents
-G agents (sarin, soman, tabun, V agent)

4. Metabolic Agents (Cyanides)
-cyanide

Categories of Chemical Agents

1. Vesicants
-primary route of exposure = skin (contact), however if vesicants remain on skin or clothing long enough they produce vapors that can enter resp. tract
-cause burn-like blisters to form on patient’s skin and in resp. tract
-cause the most damage to damp or moist areas of body (i.e. armpits, groin, and resp. tract)

Signs of vesicant exposure on skin:
-skin irritation, burning, and reddening
-immediate intense skin pain (with L and CX)
-formation of large blisters
-gray discoloration of skin (a sign of permanent damage seen with L and CX)
-swollen and closed or irritated eyes
-permanent eye injury (including blindness)

If vapors were inhaled, the patient may experience:
-hoarseness and stridor
-severe cough
-hemoptysis (coughing up blood)
-severe dyspnea

a. Sulfur mustard (H) = brownish, yellowish oily substance that is generally considered very persistent (stays on surfaces a long time)
-when released, has the smell of garlic or mustard and is quickly absorbed into skin and/or mucous membranes (within seconds)
-once absorbed into skin, begins irreversible process of damage to cells (within 1-2 minutes of absorption); eventually cellular death will occur
-considered a mutagen
-patient won’t present with signs/symptoms until 4-6 hours after exposure. Then, patient will develop a progressive reddening of affected area, which will develop into large blisters (similar to those of 2nd degree burns)
-although the fluid within the blisters does NOT contain any sulfur mustard agent, the skin is considered contaminated and must be decontaminated
-primary complication = second infection (because mustard attacks vulnerable cells in bone marrow and depletes the body’s ability to reproduce WBCs, making patient have a decreased resistance to infections)
-when vapors are inhaled, causes upper and lower airway compromise, leading to damage and swelling of the airways

b. Lewisite (L) and phosgene oxime (CX)
-produce blister wounds similar to those caused by mustard
-However, they are highly volatile and have a rapid onset of symptoms
-produce immediate intense pain and discomfort when contact is made
-patient may have grayish discoloration at contaminated site
-do not cause secondary cellular injury that mustard does, although tissue damage also occurs

Treatment:
-No antidotes for mustard or CX, but Bristish anti-Lewisite is the antidote for agent L (not carried by civilian EMS)
-ensure patient has been decontaminated before assessing ABCs
-transport asap, preferably to burn center that is best equipped to handle wounds and subsequent infections produced by vesicants

Chemical Agents — Vesicants (blister agents)

2. Pulmonary agents (choking agents)
-these are gases that cause immediate harm to exposed patients
-primary route of exposure = through respiratory tract (makes them an inhalation or vapor hazard)
-once inside lungs, they damage the lung tissue and fluid leaks into lungs; pulmonary edema develops, resulting in difficulty breathing (because of inability for air exchange)
-produce respiratory-related symptoms such as dyspnea, tachypnea, and pulmonary edema

a. Chlorine (CL) = first chemical agent ever used in warfare
-smells like bleach and creates a green haze when released as a gas
-initially produces upper airway irritation and a choking sensation
-Later, patient may experience the following signs/symptoms:
-shortness of breath
-chest tightness
-hoarseness and stridor as the result of upper airway constriction
-gasping and coughing

-with serious exposures, patient may exhibit pulmonary edema, complete airway constriction, and death
-household bleach (CL) and ammonia fumes create an acid gas that produces similar effects (common when people try to mix household cleaners)

b. Phosgene should NOT be confused for phosgene oxime
-phosgene is also a product of combustion (such as might be produced at a fire). Therefore, you may encounter a patient with exposure to this gas during a normal call at a fire scene
-potent agent that has delayed onset of symptoms (hours)
-unlike CL, it does NOT cause severe irritation that would possibly cause victim to leave the area or hold his/her breath
-smells like freshly mown grass or hay -> therefore, much more of the gas is allowed to enter body unnoticed

Initially, a mild exposure may include:
-nausea
-chest tightness
-severe cough
-dyspnea on exertion

-severe exposure patient may present with dyspnea at rest and excessive pulmonary edema (patient will expel large amounts of fluid because of this, possibly becoming hypovolemic and hypotensive)

Treatment:
-remove patient from contaminated atmosphere
-aggressive management of ABCs with focus on oxygenation, ventilation, and suctioning if required
-do NOT allow patient to be active (this will worsen patient’s condition)
-there are no antidotes for pulmonary agents
-allow patient to rest in position of comfort with head elevated and rapidly transport

Chemical agents — Pulmonary agents (choking agents)

-among the most deadly chemicals developed and are designed to kill large numbers of people with small quantities
-can cause cardiac arrest within seconds-minutes of exposure
-are a class of chemicals called organophosphates (found in household bug sprays, pesticides, and some industrial chemicals in lower strengths)
-organophosphates block an essential enzyme in the nervous system, causing the body’s organs to become overstimulated and burn out
-nerve agents have similar symptoms but varying routes of entry
-these are the only type of chemical agent that has been used successfully in terrorist act

a. G agents
-came from early nerve agents, the G series, created by a German scientist (hence the G) after WWI and into WWII
-there are 3 G agents, all designed with same basic chemical structure with two slight variations: lethality and volatility
-listed from high volatility (evaporates easily) to low volatility (evaporates slowly):
a. Sarin (GB)
b. Soman (GD)
c. Tabun (GA)
d. V agent (VX)

a. Sarin (GB) = highly volatile colorless and odorless liquid
-turns from liquid to gas within seconds-minutes
-highly lethal with an LD50 of 1,700 mg/70kg (about 1 drop)
-LD50 = amount that will kill 50% of people who are exposed to this level
-vapor hazard with respiratory tract as main route of entry
-dangerous in enclosed spaces (i.e. office buildings, etc.)
-when sarin is on clothing, it has the effect of off-gassing, which means that the vapors are continuously released over a period of time (like perfume), making victim and clothing contaminated

b. Soman (GD) = twice as persistent (stays on surfaces twice as long) as sarin and five times as lethal
-colorless and has fruity odor (due to type of alcohol used in agent)
-it’s a contact and inhalation hazard that can enter body through skin absorption and through the respiratory tract
-it binds to the cells that it attacks faster than any other agent; this irreversible binding is called aging, which makes it more difficult to treat patients who have been exposed
-aging = process by which the temporary bond between the organophosphate (nerve agent) and acetylcholinesterase (enzyme) undergoes hydrolysis, resulting in a permanent covalent bond

c. Tabun (GA) = half as lethal as sarin and 36 times more persistent
-can remain present for several days
-also has fruity smell and appearance similar to sarin
-components of this nerve agent are easy to acquire and it’s easy to manufacture
-it’s a contact and inhalation hazard that can enter body through skin absorption and respiratory tract

d. V agent (VX) = most toxic chemical ever created (small LD50)
-clear, oily agent that has no odor and looks like baby oil
-developed after WWII and has chemical properties similar to G series
-difference = VX is 100 times more lethal than sarin and is extremely persistent (can remain relatively unchanged for weeks-months)
-primarily a contact hazard because it lets off little vapor; only residue on the skin after contact is extremely difficult to decontaminate

Chemical agents — Nerve agents

V agent (VX), a nerve agent

What is the most toxic chemical ever created?

Military = SLUDGEM
Medical = DUMBELS

SLUDGEM:
Salivation, Sweating
Lacrimation (excessive tearing)
Urination
Defecation, Drooling, Diarrhea
Gastric upset and cramps
Emesis (vomiting)
Muscle twitching/Miosis (pinpoint pupils)

DUMBELS (more useful)
Diarrhea
Urination
Miosis (pinpoint pupils)
Bradycardia, Bronchospasm
Emesis (vomiting)
Lacrimation
Seizures, salivation, sweating

*seizures will continue until patient dies or until treatment is given with a nerve agent antidote kit (Mark 1 NAAK)

What mnemonics are used to describe signs/symptoms of nerve agent exposure?

cerebrovascular accident, direct light to both eyes, and drug overdose with cholinergic agents

What medical conditions have bilateral pupil constriction (miosis) seen with nerve agent exposure?

miosis (bilateral pinpoint constricted pupils)

-can remain for days to weeks

What is the most common symptom of nerve agent exposure?

respiratory complications, which lead to respiratory arrest

-therefore it’s important to provide airway and ventilatory support

Fatalities from severe nerve agent exposure occur as a result of _______.

Treatment:
-fatalities from severe nerve agent exposure occur as a result of respiratory complications, which lead to respiratory arrest
-therefore, it’s important to provide aggressive airway and ventilatory support after patient has been decontaminated
-treatment = nerve agent antidote kit; most common is the Mark 1 Nerve Agent Antidote Kit (NAAK)
-Mark 1 NAAK contains two medications: 2 mg atropine and 600 mg of pralidoxime chloride (2-PAM) in two separate auto-injectors
-an updated version of the Mark 1 is the DuoDote Auto-Injector, which contains 2.1 mg atropine and 600 mg 2-PAM and is delivered in a single dose through one needle
-multiple doses may be necessary, used same as EpiPen
-atropine is used to block the nerve agent from affecting the body (it’s an anticholinergic agent) while 2-PAM is used eliminate the agent from the body
-many of the symptoms in DUMBELS/SLUDGEM will be reversed with use of atropine, but many doses may be necessary

What is the treatment for nerve agents?

-hydrogen cyanide (AC) and cyanogen chloride (CK) are both agents that affect the body’s ability to use oxygen
-cyanide = colorless gas that has an odor similar to almonds (AC)
-effects of cyanides begin at cellular level and are rapidly seen at organ and system levels
-besides nerve agents, metabolic agents are the only chemical weapons known to kill within seconds-minutes; however, these agents are commonly found in many industrial settings (gold and silver mining, photography, plastics processing) and are often present in fires associated with textile and plastic factories
-cyanide is naturally found in very low doses in the pits of fruits
-vapor hazard
-binds to body’s cells, preventing oxygen from being used
-with low doses, signs/symptoms include dizziness, light-headedness, headache, and vomiting

Signs/Symptoms (little difference between AC and CK) of higher dose (will appear within several minutes):
-shortness of breath and gasping respirations
-tachypnea
-flushed skin
-tachycardia
-altered mental status
-seizures
-coma
-apnea (cessation of breathing)
-cardiac arrest

*death will occur unless patient is treated promptly

Treatment:
-several medications act as antidotes but most services do not carry them
-all of patient’s clothing must be removed by HazMat to prevent off-gassing in the ambulance and patient must be decontaminated before you can begin treatment
-remove from source of contamination and provide supplemental oxygen
-severe exposure will require aggressive oxygenation and perhaps ventilation with supplemental oxygen
-always use a BVM device or oxygen-powered ventilator device to ventilate a victim of a metabolic agent (because agent can be easily passed from patient to you through mouth-to-mouth or mouth-to-mask ventilations)
-if no antidote is available, initiate transport immediately

Chemical agents — metabolic agents (cyanides)

-make sure patient is thoroughly decontaminated before you come into contact with them
-all of patient’s clothing must be removed prior to you providing treatment to prevent off-gassing (chemical agents are primarily a vapor hazard)
-NEVER perform mouth-to-mouth or mouth-to-mask ventilations on a victim of a chemical agent exposure because vapors may linger in patient’s airway and cross-contamination may occur

General rules to keep in mind when treating patients exposed to chemical agents:

1. Viruses
-Smallpox
-Viral hemorrhagic fevers (VHF)

2. Bacteria
-Inhalation and Cutaneous Anthrax (Bacillus anthracis)
-Plague (Bubonic/Pneumonic)

3. Neurotoxins
-Botulinum toxin
-Ricin

-can be almost undetectable and most of the diseases caused by these agents will be similar to other minor illnesses commonly seen by EMS providers
-dissemination = means by which a terrorist will spread the agent (e.g. poisoning the water supply or aerosolizing the agent in to the air or ventilation system of a building)
-a disease vector = an animal that spreads disease, once infected, to another animal (e.g. bubonic plague can be spread by infected rats, smallpox by infected persons)
-communicability = how easily the disease is spread from one human to another human
-when communicability is high, such as with smallpox, person is considered contagious
-incubation = period of time between the person becoming exposed to the agent and when symptoms begin → important for EMTs because patient may not exhibit symptoms and yet still be contagious
-if the agent is in the form of a powder, HazMat must handle the incident

Biologic Agents

communicability

-when communicability is high, such as with smallpox, person is considered contagious

How easily the disease is spread from one human to another human

incubation

Period of time between the person becoming exposed to the agent and when symptoms begin

=germs that require a living host to multiply and survive
-as the virus spreads, so does the disease that it carries
-viruses move from host to host by direct methods, such as respiratory droplets or through vectors (act as carriers/transporters)
-although some viral agents have vaccines, there is NO treatment for a viral infection other than antiviral medications for some agents

a. Smallpox
=highly contagious disease
-all forms of standard precautions must be worn to prevent cross-contamination (i.e. examination gloves, HEPA-filtered respirator, and eye protection)
-before the rash and blisters show, the illness will start with a high fever and body aches and headaches
-patient’s temperature is usually in the range of 101 F to 104 F
-there is a vaccine to prevent smallpox but it has been linked to medical complications and possible death
-should an outbreak occur, the U.S. government has enough vaccine to vaccinate every person in the United States
-dissemination = aerosolized for warfare or terrorist uses
-communicability = high from infected individuals or items (such as blankets); person-to-person transmission is possible
-route of entry = inhalation of coughed droplets or direct skin contact with blisters
-no specific treatment for smallpox, simply provide supportive care of ABCs

Signs/Symptoms:
-severe fever
-malaise
-body aches
-headaches
-small blisters on the skin
-bleeding of skin and mucous membranes

Biologic Agents — Viruses – Smallpox

Observe the size, shape, and location of the lesions
-in smallpox, all the lesions are identical in their development, whereas in other skin disorders, the lesions will be in various stages of healing and development
-smallpox blisters also begin on the face and extremities and eventually move toward the chest and abdomen

What is one way you can distinguish/differentiate the smallpox rash from other skin disorders?

when blisters begin to form

When is smallpox in its most contagious phase?

-consist of a group of diseases caused by viruses that include the Ebola, Rift Valley, and yellow fever viruses, among others that cause the blood in the body to seep out from the tissues and blood vessels
-the end result is ecchymosis, hemoptysis, and blood in the patient’s stool
-patients’ skin will have severe discoloration, indicating internal bleeding
-initially patient will have flulike symptoms, progressing to more serious symptoms such as internal and external hemorrhaging
-all standard precautions must be taken when treating these illnesses
-mortality rates can range from 5-90% depending on the strain of virus, the victim’s age and health condition, and the availability of a modern health care system

-dissemination = direct contact with an infected person’s body fluids; can also be aerosolized for use in an attack
-communicability = moderate from person to person or contaminated items
-route of entry = direct contact with an infected person’s body fluids
-treatment = no specific treatment, provide supportive care of ABCs and treat for shock and hypotension if present

Signs/symptoms:
-sudden onset of fever
-weakness
-muscle pain
-headache and sore throat
^^all followed by vomiting, and internal and external bleeding

Biologic Agents — Viruses – Viral Hemorrhagic Fevers (VHF)

-unlike viruses, bacteria do NOT require a host to multiply and live
-bacteria contain all the cellular structures of a normal cell and are completely self-sufficient
-most bacterial infections can be fought with antibiotics and generally begin with flulike symptoms, which can make it quite difficult for providers to identify whether the cause is a biologic attack or a natural epidemic

a. Inhalation and Cutaneous Anthrax (Bacillus anthracis)
-anthrax is caused by a deadly bacterium that lays dormant in a spore. When exposed to the optimal temperature and moisture, the germ will be released from spore
-routes of entry for anthrax are inhalation, cutaneous, and gastrointestinal (from consuming food that contains spores)
-inhalation form (pulmonary anthrax) is the most deadly and often presents with a severe cold (has 90% death rate if untreated)
-antibiotics can be used to treat anthrax successfully, and there is also a vaccine to prevent anthrax infections

-dissemination = aerosol
-communicability = only in the cutaneous form (rare)
-route of entry = through inhalation of spore or skin contact with spores or direct contact with skin wound (cutaneous)
-treatment for pulmonary anthrax = oxygen, ventilatory support if in pulmonary edema or respiratory failure, and transport
-treatment for cutaneous anthrax = apply dry sterile dressing to prevent accidental contact with wound and fluids

Signs/symptoms:
-flulike symptoms
-fever
-respiratory distress with tachycardia, shock, pulmonary edema, and respiratory failure after 3-5 days of flulike symptoms

Biologic Agents — Bacteria – Anthrax

-plague’s natural vectors are infected rodents and fleas
-when a person is bitten by an infected flea or comes into contact with an infected rodent (or waste of rodent), the person can contract bubonic plague

-Bubonic plague (Black Death) infects the lymphatic system (a passive circulatory system in the body that bathes the tissues in lymph and works with the immune system)
-when this occurs, the patient’s lymph nodes (area of lymphatic system where infection-fighting cells are housed) become infected and grow. The glands of the nodes will grow large (up to the size of a tennis ball) and round, forming buboes
-buboes = enlarged lymph nodes that wre characteristic in people infected with bubonic plague
-if left untreated, infection may spread through the body, leading to sepsis and possibly death
-this form of plague is NOT contagious and is not likely to be seen in a bioterrorist incident
-characterized by acute malaise, fever, and the formation of tender, enlarged, inflamed lymph nodes that appear as lesions, called buboes

-Pneumonic plague = a lung infection, also known as plague pneumonia, that results from inhalation of plague bacteria
-this form of the disease is contagious and has a much higher death rate than the bubonic form

-dissemination = aerosol
-communicability:
-bubonic = low, only from contact with fluid in buboes
-pneumonic = high, from person to person
-route of entry = ingestion, inhalation, or cutaneous
-treatment = ABCs, provide oxygen, and transport

Signs/symptoms:
-fever, headache, muscle pain and tenderness
-pneumonia
-shortness of breath
-extreme lymph node pain and enlargement (bubonic)

Biologic Agents — Bacteria – Plague (Bubonic/Pneumonic)

pneumonic plauge is contagius and has much higher death rate than bubonic form

Which form of the plague is contagious?

neurotoxins

What are the most deadly substances known to humans?

= most deadly substances known to humans
-strongest neurotoxin is 15,000 times more lethal than VX (most toxic chemical) and 100,000 more lethal than sarin
-these toxins are produced from plants, marine animals, molds, and bacteria
-route of entry for these toxins is through ingestion, inhalation from aerosols, or injection
-unlike viruses and bacteria, neurotoxins are NOT contagious and have a faster onset of symptoms
-have not yet been used as WMD even though they have immense destructive potential

a. Botulinum toxin = the most potent neurotoxin
-produced by bacteria
-when introduced into body, this neurotoxin affects the nervous system’s ability to function. Voluntary muscle control diminishes as the toxin spreads. Eventually the toxin causes muscle paralysis that begins at the head and face and travels downward throughout the body
-the patient’s accessory muscles and diaphragm will become paralyzed and the patient will go into respiratory arrest

-dissemination = aerosol or food supply sabotage or injection
-communicability = none (not contagious)
-route of entry = ingestion, inhalation
-treatment = ABCs, provide oxygen, and transport. Ventilatory support in case of paralysis of the respiratory muscles; vaccine is available

Signs/symptoms:
-dry mouth
-intestinal obstruction
-urinary retention
-constipation
-nausea and vomiting
-abnormal pupil dilation, blurred vision, double vision, drooping eyelids
-difficulty swallowing
-difficulty speaking
-respiratory failure as the result of paralysis

Biologic Agents — Neurotoxins – Botulinum toxin

-while not as deadly as botulinum, ricin is still five times more lethal than VX
-derived from mash that is left from the castor bean. These seemingly harmless castor beans contain the key ingredient for ricin, one of the most potent toxins known to humans
-when introduced to the body, ricin causes pulmonary edema and respiratory and circulatory failure, leading to death
-the clinical picture depends on the route of exposure (ingestion vs. inhalation)
-toxin is quite stable and extremely toxic by many routes of exposure, including inhalation
-it is likely that 1 to 3 mg of ricin can kill an adult, and the ingestion of one seed can most likely kill a child
-although all parts of the castor bean are poisonous, it is the seeds that are the most toxic
-castor bean ingestion causes a rapid onset of nausea, vomiting, abdominal cramps, and severe diarrhea, followed by vascular collapse. Death usually occurs on the third day in the absence of medical intervention

-ricin is LEAST toxic by the oral route, most likely as a result of poor absorption in the GI tract, some digestion in the gut, and possibly some expulsion of the agent as caused by the rapid onset of vomiting
-ingestion causes local hemorrhage and necrosis (cell death) of the liver, spleen, kidneys, and GI tract. Signs and symptoms appear 4-8 hours after exposure

Signs/symptoms:
-fever, chills, headache
-muscle aches
-nausea, vomiting
-diarrhea
-severe abdominal cramping
-dehydration
-GI bleeding
-necrosis of liver, spleen, kidneys, and GI tract

-inhalation of ricin causes nonspecific weakness, cough, fever, hypothermia, and hypotension
-symptoms occur 4-8 hours after inhalation, depending on the inhaled dose
-the onset of profuse sweating some hours later signifies the termination of the symptoms

Signs/symptoms of ricin inhalation:
-fever, chills
-nausea
-local irritation of eyes, nose, throat
-profuse sweating
-headache, muscle aches
-nonproductive cough
-chest pain
-dyspnea
-pulmonary edema
-severe lung inflammation
-cyanosis
-seizures
-respiratory failure

Biologic Agents — Neurotoxin – Ricin

1 to 3 mg of ricin can kill an adult, and the ingestion of one seed can most likely kill a child

How much ricin neurotoxin (derived from the mash that is left from the castor bean) would it take to kill an adult?

seeds

Although all parts of the castor bean are poisonous, it is the ____ that are the most toxic

-ricin is LEAST toxic by the oral route, most likely as a result of poor absorption in the GI tract, some digestion in the gut, and possibly some expulsion of the agent as caused by the rapid onset of vomiting
-ingestion causes local hemorrhage and necrosis (cell death) of the liver, spleen, kidneys, and GI tract. Signs and symptoms appear 4-8 hours after exposure

Signs/symptoms:
-fever, chills, headache
-muscle aches
-nausea, vomiting
-diarrhea
-severe abdominal cramping
-dehydration
-GI bleeding
-necrosis of liver, spleen, kidneys, and GI tract

Through what route is ricin least toxic? Signs/symptoms?

profuse sweating

The onset of _____ some hours later after inhalation of ricin neurotixin signifies the termination of the symptoms.

In both, symptoms appear 4-8 hours after exposure

Inhalation:
-cough, difficulty breathing, chest tightness, nausea, muscle aches, pulmonary edema, and hypoxia

Ingestion:
-nausea and vomiting, internal bleeding, and death

Injection:
-no signs except swelling at the injection site and death

Treatment is supportive and includes both respiratory support and cardiovascular support as needed
-early inubation and ventilation, combined with treatment of pulmonary edema are appropriate
-IV fluids and electrolyte replacement are useful for treating the dehydration caused by profound vomiting and diarrhea

Signs and symptoms of inhaled ricin vs. ingested ricin vs. injected ricin?

-syndromic surveillance
-points of distribution (strategic national stockpile)

Other EMT roles during a biologic event

syndromic surveillance

-patients with signs and symptoms that resemble influenza are particularly important
-local and state health departments monitor for an unusual influx of patients with these symptoms in hopes of discovering an outbreak early
-the EMS role in syndromic surveillance is a small one yet valuable in overall tracking of a biologic terrorist event or infectious disease outbreak
-quality assurance and dispatch operations need to be aware of an unusual number of calls from patients with "unexplainable flu" coming from a particular region or community

The monitoring, usually by local or state health departments, of patients presenting to emergency departments and alternative care facilities, the recording of EMS call volume, and the use of over-the-counter medications

points of distribution (PODs)

-these medications may be delivered in large containers known as "push packs" by CDC and National Pharmaceutical Stockpile –> they can deliver one of many push packs to any location in the country within 12 hours of an emergency
^^push packs contain antibiotics, chemical antidotes, antitoxins, and life-support medications, IV administration supplies, airway maintenance supplies, and medical/surgical items
-in some regions, local and state municipalities have started to stockpile their own supplies to reduce the time dealy
-EMS workers may be called on to assist in delivery of the medications to the public
-your role may include triage, treatment of seriously ill patients, and patient transport to the hospital
-most plans for PODs include at least one ambulance on standby for the transport of seriously ill patients

Existing facilities that are established in a time of need for the mass distribution of antibiotics, antidotes, vaccinations, and other medications and supplies

ionizing radiation

-this energy can be found in radioactive material, such as rocks and metals
-radioactive material = any material that emits radiation. This material is unstable and it attempts to stabilize itself by changing its structure in a natural process called decay
-As substance decays, it gives off radiation until it stabilizes (process can take minutes to billions of years, all the while the substance remains radioactive

-the energy that is emitted from a strong radiologic source is alpha, beta, gamma (x-ray), or neutron radiation

The energy that is emitted in the form of rays or particles

Alpha Radiation
-least harmful penetrating type of radiation
-cannot move through most objects (sheet of paper on skin stops it)

Beta Radiation
-slightly more penetrating
-requires a layer of clothing to stop it

Gamma rays (x-rays)
-far faster and stronger than alpha and beta
-can easily penetrate through the human body and require lead or several inches of concrete to prevent penetration

Neutron particles
-one of most powerful forms of radiation
-easily penetrate through lead and require several feet of concrete to stop them

What are the types of ionizing radiation (the energy that is emitted from a strong radiologic source)?

-radiologic materials are generally used for purposes that benefit mankind, such as medications, killing germs in food (irradiating), and construction work
-once radiologic material has been used for these purposes, it’s called radiologic waste, and this remains radioactive but has no more usefulness

These materials can be found at:
-hospitals
-colleges and universities
-chemical and industrial sites

-not all radioactive material is tightly guarded and the waste is often not guarded (appealing to terrorists)

What are some sources of radiologic material?

radiologic dispersal device (RDD) –> generally requires use of a bomb

-"dirty bomb" = name given to a bomb that is used as a radiologic dispersal device; disperses radioactive material as well as explosive material
-destructive ability of a dirty bomb is limited to the explosives that are attached to it
-may be long term injuries and illnesses associated with the use of an RDD, yet not much more than the bomb by itself would create (without the radioactive material)

overall, the dirty bomb is an effective WMD

Any container that is designed to disperse radioactive material

nuclear energy

-result is an immense amount of energy that usually takes the form of heat
-nuclear material is used in medicine, weapons, naval vessels, and power plants
-nuclear material gives off all forms of radiation, including neutrons (most deadly type)
-like radioactive material, when nuclear material is no longer useful it becomes waste that is still radioactive

What is artificially made by altering (splitting) radioactive atoms?

special atomic demolition munitions (SADM) (also called "suitcase nuke")

-due to collapse of Soviet Union, the whereabouts of many of these is unknown
-estimate ~80 are missing

-nuclear weapons are very bad (think mutual annihilation)

Small suitcase-sized nuclear weapons that were designed to destroy individual targets, such as important buildings, bridges, tunnels, and large ships

True

-patient can inhale radioactive dust or have radioactive liquid absorbed into the body through the skin
-once in the body, the radiation source will irradiate the person from within rather than from an external source (such as x-ray equipment)

-the effects of radiation exposure will vary depending on the amount of radiation that a person receives and the route of entry

True or False: Radiation can be introduced into the body by all routes of entry as well as through the body (irradiation)

Low exposure
-nausea, vomiting, diarrhea

Moderate exposure
-first-degree burns
-hair loss
-depletion of the immune system (death of WBCs)
-cancer

Severe exposure
-second- and third-degree burns
-cancer
-death

What are some common signs of acute radiation sickness?

-being exposed to radiation –> patient is not contaminated or radioactive
-however, when patients have radioactive source on their body (such as debris from a dirty bomb), they are contaminated and must be initially cared for by a HazMat responder
-once patient is decontaminated and there is no threat to you, you may begin treatment with the ABCs and treat patient for any burns or trauma

Medical management of patient exposed to radiation/radioactive material

-there are NO suits or protective gear designed to completely shield you from radiation
-people who work in high-risk areas wear some protection (lead-lined suits)
-best way to protect yourself is to use time and distance and shield yourself using buildings and walls for protection

Time
-radiation has cumulative effect on body (less time you’re exposed, less the effects will be)

Distance
-radiation is limited to how far it can travel
-alpha radiation can only travel a few inches, whereas gamma rays can travel for hundreds-thousands of meters
-make certain that responders are stationed far enough from the incident

Shielding
-always assume you are dealing with the strongest form of radiation and use concrete shielding (such as buildings or walls) between yourself and the incident

Protective Measures when responding to a nuclear or radiologic incident

-come in various shapes and sizes (includes IEDs)
-need to be able to identify an object you believe is a potential device, notify proper authorities, and safely evacuate the area

Incendiary and explosive devices

-blast injuries from incendiary and explosive devices can occur in a number of ways

Primary blast injury
-due solely to the direct effects of the pressure wave on the body
-seen almost exclusively in the hollow organs of the body (i.e. lungs, intestines, and inner ears)
-injury to lungs causes greatest morbidity and mortality

Secondary blast injury
-penetrating or nonpenetrating injury that results from being struck by flying debris
-objects are propelled by the force of the blast and strike the victim, causing injury

Tertiary blast injury
-results from whole body displacement and subsequent traumatic impact with environmental objects (e.g. trees, buildings, and vehicles)
-other indirect effects include crush injury from collapse of structures and toxic effects from the inhalation of combustion gases

Mechanisms of Injury

-when a substance is detonated, a solid or liquid is chemically converted into large volumes of gas under high pressure with resultant explosive energy release

-propellants like gunpowder are explosives designed to release energy slowly compared to high energy explosives which are designed to detonate very quickly

-this generates a pressure pulse in the shape of a spherical blast wave that expands in all directions from the point of explosion

The physics of an explosion

-hollow organs such as middle ear, lung, and GI tract are most susceptible to pressure changes
-junction between tissues of different densities and exposed tissues such as head and neck are also prone to injury

Tissues at risk

ear

What organ is most sensitive to blast injuries?

primary pulmonary blast injuries –> occur as contusions and hemorrhages

-patient may complain of tightness in chest or pain in chest
-may cough up blood and have tachypnea or other signs of resp. distress
-subcutaneous emphysema over chest can be palpated, indicating air in the thorax
-pneumothorax is common and may require emergency needle decompression

-subarachnoid and subdural hematomas are common
-permanent or transient neurologic deficits may be secondary to concussion, intracerebral bleeding, or air embolism
-instant but transient unconsciousness, with or without retrograde amnesia, may be initiated not only with head trauma but also by cardiovascular problems
-bradycardia and hypotension are common after an intense pressure wave from an explosion –> this is a vagal nerve-mediated form of cardiogenic shock without compensatory vasoconstriction (for example, vasovagal syncope)
-extremity injures are common and are often associated with tertiary blasts
-patients with traumatic amputation are likely to sustain fatal injuries secondary to the blast

Pulmonary trauma resulting from short-range exposure to the detonation of high energy explosives

petechiae (pinpoint hemorrhages) to large hematomas

What is the most visible sign of damage/injury to hollow organs following a blast/explosion?

neurologic injuries and head trauma

-subarachnoid and subdural hematomas are common

What are the most common causes of death from blast injuries?

The Centennial Park bombing during the 1996 Summer Olympics is an example of:

Domestic terrorism.

A weapon of mass destruction (WMD) is MOST accurately defined as:

Any agent used to bring about mass death, casualties, or massive infrastructural damage.

Most terrorist attacks are:

Covert.

In determining the potential for a terrorist attack, you should routinely observe all of the following on every call, EXCEPT:

Weather conditions.

Cross-contamination occurs when:

An EMT is exposed to a victim who has not yet been decontaminated.

Which of the following agents blocks the body’s ability to use oxygen and possesses an odor similar to almonds?

Hydrogen cyanide

The means by which a terrorist will spread a particular agent is called:

Dissemination.

Early signs and symptoms of smallpox include all of the following, EXCEPT:

Skin blisters.

When introduced into the body, ricin causes:

Pulmonary edema and circulatory failure.

The LEAST harmful form of ionizing radiation is:

Alpha.

__________ rays easily penetrate through the human body and require several inches of lead or concrete to prevent penetration.

Gamma

A 52-year-old male presents with a fever of 102.5°F and a severe headache. As you assess him, you note the presence of multiple blisters on his face and chest, which are all identical in shape and size. This patient’s clinical presentation is MOST consistent with:

smallpox.

A disease vector is defined as:

any agent that acts as a carrier or transporter.

A severe risk of terrorist attacks is indicated by the color _____ by the Department of Homeland Security Security Advisory System.

red

After eating at a local restaurant, a 20-year-old male complains of blurred vision, difficulty speaking, and difficulty breathing. He is conscious; however, his respirations are profoundly labored and producing minimal tidal volume. You should:

assist his ventilations with 100% oxygen.

As you and your partner report for duty, you check your ambulance and begin talking about the possibility of a terrorist attack. The MOST effective and appropriate way to determine the likelihood of this happening is to:

know the current threat level issued by the Department of Homeland Security.

At present, the likelihood of a nuclear attack against the United States is very low because:

terrorist nations do not have the ability to deliver a nuclear weapon via missile or bomb.

Exposure to _________ would MOST likely result in immediate respiratory distress.

chlorine

If the incident command system (ICS) is already established at the scene of a WMD or terrorist attack, the EMT should:

locate the medical staging officer to obtain his or her assignment.

Most terrorist attacks:

require multiple terrorists working together.

Nerve agents, a class of chemicals called organophosphates, were first discovered while in search of a superior:

pesticide.

Points of distribution (PODs) are strategically placed facilities where:

antidotes, antibiotics, and vaccines are distributed.

Signs and symptoms of exposure to a nerve agent include:

salivation, pinpoint pupils, and diarrhea.

Symptoms of both inhaled and ingested ricin include:

fever and headache.

The chemical attacks that occurred in Tokyo between 1994 and 1995 were carried out by a(n):

violent religious group.

The primary clinical feature associated with exposure to phosgene oxime is:

skin blistering.

The process performed to artificially maximize the target population’s exposure to a biologic agent, thereby exposing the greatest number of people and achieving the desired effect, is called:

weaponization.

The type and severity of wounds sustained from incendiary and explosive devices primarily depend on the:

patient’s distance from the epicenter of the explosion.

Unlike viruses and bacteria, neurotoxins:

are not contagious.

Which of the following statements regarding blast injuries is correct?

Solid organs are relatively protected from shock wave injury but may be injured during the secondary or tertiary blast phase.

You are dispatched to the scene of a building explosion. Upon arrival, you see people frantically fleeing the building, screaming, "Everyone is passing out!" You should:

carefully assess the situation and ensure your own safety.

__________ rays easily penetrate through the human body and require lead or several inches of concrete to prevent penetration.

Gamma

After eating at a local restaurant, a 20-year-old male complains of blurred vision, difficulty speaking, and difficulty breathing. He is conscious; however, his respirations are profoundly labored and producing minimal tidal volume. You should:

assist his ventilations with high-flow oxygen

All of the following are vesicant agents, EXCEPT:

sarin

An attack on an abortion clinic would MOST likely be carried out by a(n):

C. single-issue group.

As you and your partner report for duty, you check your ambulance and begin talking about the possibility of a terrorist attack. The MOST effective and appropriate way to determine the likelihood of this happening is to:

D. know the current threat level issued by the Department of Homeland Security.

At present, the likelihood of a nuclear attack against the United States is very low because:

A. terrorist nations do not have the ability to deliver a nuclear weapon via missile or bomb.

Continual reassessment of the scene at a suspected terrorist or weapon of mass destruction incident is MOST important because:

D. a secondary explosive device may detonate.

In determining the potential for a terrorist attack, you should routinely observe all of the following on every call, EXCEPT:

B. weather conditions.

Most cases of anthrax begin with:

B. flulike symptoms.

Multiple people in a small town began experiencing abdominal cramps, excessive salivation and urination, and muscle twitching shortly after a small crop duster plane made several passes over the community. As you are assessing the patients, you further determine that most of them are bradycardic and have miosis. In addition to high-flow oxygen, the MOST appropriate treatment for these patients includes:

D. atropine and pralidoxime chloride.

Points of distribution (PODs) are strategically placed facilities where:

B. antidotes, antibiotics, and vaccines are distributed.

The Centennial Park bombing during the 1996 Summer Olympics is an example of:

C. domestic terrorism.

The EMT should expect that a patient who was exposed to cyanide will have:

B. a normal pulse oximetry reading.

The incubation period for Ebola can be up to:

C. 21 days.

The means by which a terrorist will spread a particular agent is called:

dissemination

The type and severity of wounds sustained from incendiary and explosive devices primarily depend on the:

A. patient’s distance from the epicenter of the explosion.

To date, the preferred weapons of mass destruction for terrorists have been:

A. explosive weapons.

Unlike viral agents, bacterial agents:

B. respond to antibiotics.

Which of the following statements regarding blast injuries is correct?

C. Solid organs are relatively protected from shock wave injury but may be injured during the secondary or tertiary blast phase.

You and your partner arrive at the scene of a fire at a large office complex. Witnesses tell you that they heard a loud explosion shortly before the building caught fire. You should:

A. ensure that your ambulance is parked upwind and uphill from the building.

The chemical attacks that occurred in Tokyo between 1994 and 1995 were carried out by a(n):

violent religious group.

Which of the following terrorist groups poses the LEAST threat to a person’s physical safety?

Cyber terrorists

A weapon of mass destruction is MOST accurately defined as:

any agent used to bring about mass death, casualties, or massive infrastructural damage.

The process performed to artificially maximize the target population’s exposure to a biologic agent, thereby exposing the greatest number of people and achieving the desired result, is called:

weaponization.

Most terrorist attacks are:

covert

You are dispatched to the scene of a building explosion. Upon arrival, you see people frantically fleeing the building, screaming, "Everyone is passing out!" You should:

carefully assess the situation and ensure your own safety.

Cross-contamination occurs when:

an EMT is exposed to a victim who has not yet been decontaminated.

As the first-arriving emergency responder at the scene of a suspected terrorist or weapon of mass destruction incident, you should request additional resources as needed and then:

function as the incident commander until additional personnel arrive.

A persistent or nonvolatile chemical agent can:

remain on a surface for more than 24 hours.

Exposure to _________ would MOST likely result in immediate respiratory distress.

chlorine

Signs and symptoms of exposure to a nerve agent include:

salivation, pinpoint pupils, and diarrhea.

You are treating a patient who experienced a significant exposure to cyanide. He is semiconscious and is breathing inadequately. The MOST appropriate method of providing assisted ventilations to this patient is to:

use a bag-valve mask.

Early signs and symptoms of smallpox include all of the following, EXCEPT:

skin blisters.

You are assessing a 30-year-old female who presents with respiratory distress and tachycardia after she opened a package that was delivered to her home. The patient tells you that there was a fine white powder on the package, but she did not think it was important. This patient has MOST likely been exposed to:

anthrax

When introduced into the body, ricin causes:

pulmonary edema and circulatory failure.

The LEAST harmful form of ionizing radiation is:

alpha

Which of the following is NOT a factor in determining how to protect oneself against the effects of radiation?

Body size

Pulmonary hemorrhage and inner ear damage are examples of __________ blast injuries.

primary

Which of the following would you expect to encounter in a patient with moderate radiation toxicity?

Hair loss and first-degree burns

Unlike viruses and bacteria, neurotoxins:

are not contagious.

international terrorism

Terrorism that is carried out by people in a country other than their own; also known as cross-border terrorism.

domestic terrorism

Terrorism that is carried out by people in their own country

weapon of mass destruction (WMD)

Any agent designed to bring about mass death, casualties, and/or massive damage to property and infrastructure (bridges, tunnels, airports, and seaports); also known as a weapon of mass casualty (WMC).

weapon of mass casualty (WMC)

Any agent designed to bring about mass death, casualties, and/or massive damage to property and infrastructure (bridges, tunnels, airports, and seaports); also known as a weapon of mass destruction (WMD).

B-NICE

A memory device to recall the types of weapons of mass destruction: biologic, nuclear, incendiary, chemical, and explosive.

weaponization

The creation of a weapon from a biologic agent generally found in nature and that causes disease; the agent is cultivated, synthesized, and/or mutated to maximize the target population’s exposure to the germ.

state-sponsored terrorism

Terrorism that is funded and/or supported by nations that hold close ties with terrorist groups.

covert

An act in which the public safety community generally has no prior knowledge of the time, location, or nature of the attack.

secondary device

A secondary explosive used by terrorists, set to explode after the initial bomb.

cross-contamination

Occurs when a person is contaminated by an agent as a result of coming into contact with another contaminated person.

persistency

Describes how long a chemical agent will stay on a surface before it evaporates.

volatility

How long a chemical agent will stay on a surface before it evaporates.

route of exposure

The manner by which a toxic substance enters the body.

vapor hazard

The term used to describe danger posed by an agent that enters the body through the respiratory tract.

contact hazard

The term used to describe danger posed by a chemical whose primary route of entry into the body is through the skin; posed by a hazardous agent that gives off very little or no vapors; also called a skin hazard.

vesicants

Blister agents; the primary route of entry for this agent is through the skin.

sulfur mustard (H)

A vesicant; it is a brownish, yellowish oily substance that is generally considered very persistent; has the distinct smell of garlic or mustard and, when released, is quickly absorbed into the skin and/or mucous membranes and begins an irreversible process of damaging the cells. Also called mustard gas.

mutagen

A substance that mutates, damages, and changes the structures of DNA in the body’s cells.

lewisite (L)

A blistering agent that has a rapid onset of symptoms and produces immediate, intense pain and discomfort on contact.

phosgene oxime (CX)

A blistering agent that has a rapid onset of symptoms and produces immediate, intense pain and discomfort on contact.

chlorine (Cl)

The first chemical agent ever used in warfare. It has a distinct odor of bleach and creates a green haze when released as a gas. Initially it produces upper airway irritation and a choking sensation.

phosgene

A pulmonary agent that is a product of combustion, resulting from a fire at a textile factory or house or from metalwork or burning Freon. It is a very potent agent that has a delayed onset of symptoms, usually hours.

nerve agents

A class of chemical called organophosphates; they function by blocking an essential enzyme in the nervous system, which causes the body’s organs to become overstimulated and burn out.

DuoDote Auto-Injector

A nerve agent antidote kit containing atropine and pralidoxime chloride; delivered as a single dose through one needle.

Antidote Treatment Nerve Agent Auto-Injector (ATNAA)

A nerve agent antidote kit containing atropine and pralidoxime chloride; delivered as a single dose through one needle.

cyanides

An agent that affects the body’s ability to use oxygen. It is a colorless gas that has an odor similar to almonds. The effects begin on the cellular level and are very rapidly seen at the organ and system levels.

When ________ is on clothing, it has the effect of off-gassing, which renders the victim and the victim’s clothing contaminated.

Sarin(GB)

dissemination

The means by which a terrorist will spread an agent, for example, by poisoning the water supply or aerosolizing the agent into the air or ventilation system of a building.

disease vector

An animal that spreads a disease, once infected, to another animal.

incubation

The period of time between a person being exposed to an agent to the first time when symptoms appear.

smallpox

A highly contagious disease; it is most contagious when blisters begin to form.

viral hemorrhagic fevers (VHF)

A group of diseases caused by viruses that include the Ebola, Rift Valley, and yellow fevers, among others. This group of viruses causes the blood in the body to seep out from the tissues and blood vessels.

Anthrax

is caused by deadly bacteria that lay dormant in a spore (protective shell).

bubonic plague

Bacterial infection that affects the lymphatic system. It is transmitted by infected rodents and fleas and characterized by acute malaise, fever, and the formation of tender, enlarged, inflamed lymph nodes that appear as lesions, called buboes. Also called the Black Death.

pneumonic plague

A lung infection, also known as plague pneumonia, that is the result of inhalation of plague-causing bacteria.

neurotoxin

Biologic agents that are the most deadly substances known to humans; they include botulinum toxin and ricin.

The most potent neurotoxin is _________, which is produced by bacteria.

botulinum

ricin

A neurotoxin derived from mash that is left from the castor bean; causes pulmonary edema and respiratory and circulatory failure leading to death.

Syndromic surveillance

The monitoring, usually by local or state health departments, of patients presenting to emergency departments and alternative care facilities, the recording of EMS call volume, and the use of over-the-counter medications.

Points of distribution (PODs)

Existing facilities used as mass distribution sites for antibiotics, antidotes, vaccinations, and other medications and supplies during an emergency.

ionizing radiation

Energy that is emitted in the form of rays, or particles.

decay

A natural process in which a material that is unstable attempts to stabilize itself by changing its structure.

Alpha Radiation

-The least harmful penetrating type of radiation -Cannot penetrate through most objects (a sheet of paper or the body’s skin can easily stop it)

Beta radiation

Slightly more penetrating than alpha and requires a layer of clothing to stop it

Gamma (x-ray) radiation

These rays easily penetrate through the human body and require lead or several inches of concrete to prevent penetration.

Neutron radiation

-Neutron particles are among the most powerful forms of radiation. -Neutrons easily penetrate through lead and require several feet of concrete to stop them.

radiologic dispersal device (RDD)

Any container that is designed to disperse radioactive material.

dirty bomb

Name given to an explosive radiologic dispersal device.

Special Atomic Demolition Munitions (SADM)

Small suitcase-sized nuclear weapons that were designed to destroy individual targets, such as important buildings, bridges, tunnels, and large ships.

pulmonary blast injuries

Pulmonary trauma resulting from short-range exposure to the detonation of high-energy explosives.

What is a mustard gas considered to be?

Mutagen

In which group of agents is miosis a sign of exposure?

nerve agent

Which of the following requires a host to survive?

small pox

Which is the most deadly route of entry for anthrax?

inhalation

Which of the following is the means by which an agent is spread?

Dissemination

Which of the following is true regarding a dirty bomb?

The destructive capability is limited to the explosives that are attached to it

Which type of blast injury is most likely to produce penetrating trauma?

Secondary

You are on scene at a suspected terror attack in which a RDD has detonated. A 67-year-old patient reports tightness in his chest. The patient is coughing up blood and is in respiratory distress. During the physical assessment, you notice subcutaneous emphysema. What is the condition most likely to be?

Pulmonary blast injury

__________ rays easily penetrate through the human body and require lead or several inches of concrete to prevent penetration.

A. Neutron B. Beta C. Gamma Correct D. Alpha

After eating at a local restaurant, a 20-year-old male complains of blurred vision, difficulty speaking, and difficulty breathing. He is conscious; however, his respirations are profoundly labored and producing minimal tidal volume. You should:

A. position him supine and elevate his legs. B. apply oxygen via a nonrebreathing mask. C. assist his ventilations with high-flow oxygen. Correct D. request a paramedic to administer atropine.

All of the following are vesicant agents, EXCEPT:

A. phosgene oxime. B. sarin. Correct C. lewisite. D. sulfur mustard

An attack on an abortion clinic would MOST likely be carried out by a(n):

A. violent religious group. B. extremist political group. C. doomsday cult. D. single-issue group. Correct

As you and your partner report for duty, you check your ambulance and begin talking about the possibility of a terrorist attack. The MOST effective and appropriate way to determine the likelihood of this happening is to:

A. ask your immediate supervisor if he or she has been watching the local news. B. know the current threat level issued by the Department of Homeland Security. Correct C. ascertain the current situation overseas with regard to the number of casualties. D. check with local businesses to see if they have received any terrorist threats.

At present, the likelihood of a nuclear attack against the United States is very low because:

A. terrorist nations do not have the ability to deliver a nuclear weapon via missile or bomb. Correct B. other than the United States, no other countries are currently in possession of nuclear weapons. C. the United States has an effective early warning system to detect an incoming nuclear missile. D. all nuclear devices or weapons that different countries are in possession of are currently accounted for.

Continual reassessment of the scene at a suspected terrorist or weapon of mass destruction incident is MOST important because:

A. weather conditions may change quickly. B. bystanders may destroy the evidence. C. a secondary explosive device may detonate. Correct D. terrorists are often at the scene after an attack.

n determining the potential for a terrorist attack, you should routinely observe all of the following on every call, EXCEPT:

A. victim’s statements. B. weather conditions. Correct C. the location type. D. the type of call.

Most cases of anthrax begin with:

A. respiratory distress. B. signs of shock. C. pulmonary edema. D. flulike symptoms. Correct

Multiple people in a small town began experiencing abdominal cramps, excessive salivation and urination, and muscle twitching shortly after a small crop duster plane made several passes over the community. As you are assessing the patients, you further determine that most of them are bradycardic and have miosis. In addition to high-flow oxygen, the MOST appropriate treatment for these patients includes:

A. activated charcoal and glucose. B. amyl nitrate and naloxone. C. atropine and pralidoxime chloride. Correct D. epinephrine and hyperbaric oxygen.

Points of distribution (PODs) are strategically placed facilities where:

A. chemical and biologic weapons are stockpiled. B. antidotes, antibiotics, and vaccines are distributed. Correct C. weapons of mass destruction are distributed. D. chemical weapons are manufactured and distributed.

The Centennial Park bombing during the 1996 Summer Olympics is an example of:

A. state-sponsored terrorism. B. domestic terrorism. Correct C. an ecoterrorist attack. D. apocalyptic violence.

The EMT should expect that a patient who was exposed to cyanide will have:

A. an abnormally slow pulse rate. B. an abnormally slow respiratory rate. C. skin that is cherry red and hot. D. a normal pulse oximetry reading. Correct

The incubation period for Ebola can be up to:

A. 72 hours. B. 1 week. C. 6 months. D. 21 days. Correct

The means by which a terrorist will spread a particular agent is called:

A. incubation. B. dissemination. Correct C. weaponization. D. aerosolization

The type and severity of wounds sustained from incendiary and explosive devices primarily depend on the:

A. type of material used to manufacture the device. B. pressure that is generated from the explosion itself. C. size of the structure that was involved in the explosion. D. patient’s distance from the epicenter of the explosion. Correct

To date, the preferred weapons of mass destruction for terrorists have been:

A. chemical weapons. B. explosive weapons. Correct C. biologic weapons. D. nuclear weapons.

Unlike viral agents, bacterial agents:

A. are far less infectious. B. do not replicate in the body. C. are usually not treatable. D. respond to antibiotics. Correct

Which of the following statements regarding blast injuries is correct?

A. The gastrointestinal tract is the organ system most sensitive to blast injuries and is the leading cause of death following an explosion. B. Tertiary blast injuries are penetrating or nonpenetrating injuries that result from flying debris, such as ordnance projectiles. C. Solid organs are relatively protected from shock wave injury but may be injured during the secondary or tertiary blast phase. Correct D. Solid organs such as the middle ear, lungs, and gastrointestinal tract are the most susceptible to pressure changes.

You and your partner arrive at the scene of a fire at a large office complex. Witnesses tell you that they heard a loud explosion shortly before the building caught fire. You should:

A. tell the witnesses that you suspect that the explosion was the work of a terrorist. B. take standard precautions and begin searching for critically injured patients. C. carefully document the witnesses’ statements and report them immediately. D. ensure that your ambulance is parked upwind and uphill from the building. Correct

It is not uncommon for young females who experience their first menstrual period to:
A) Become so emotionally distraught that they contemplate suicide.

B) Experience abdominal cramping, which may be misinterpreted.

C) Lose up to 500 mL of blood within the first 24 hours.

D) Have a falsely positive home pregnancy test result.

B

Which of the following statements regarding pelvic inflammatory disease (PID) is correct?
A) The most severe cases of PID occur in women who are not sexually active.

B) The most common presenting symptom of PID is generalized upper abdominal pain.

C) PID most commonly affects women who have had an ectopic pregnancy in the past.

D) PID can scar the fallopian tubes, which increases the risk of an ectopic pregnancy.

D

Which of the following conditions does NOT typically present with vaginal discharge?
A) Chlamydia

B) Genital herpes

C) Gonorrhea

D) PID

B

As a woman approaches menopause:
A) She usually experiences abdominal cramping without vaginal bleeding.

B) Her menstrual periods may become irregular and vary in severity.

C) She cannot become pregnant because of fluctuating hormone levels.

D) Her risk of developing PID lowers significantly.

B

Whenever possible, a female sexual assault victim should be:
A) Thoroughly assessed, even if no signs of injury exist.

B) Asked to provide a brief description of the perpetrator.

C) Encouraged to take a shower and change her clothes.

D) Given the option of being treated by a female EMT.

D

During your assessment of a young female with nontraumatic vaginal bleeding, you note that her level of consciousness is decreased, her respirations are rapid and shallow, her skin is cool and moist, and her pulse is rapid and weak. You should:
A) Perform a visual assessment of her vaginal area.

B) Perform a rapid secondary assessment.

C) Assess her blood pressure and elevate her legs.

D) Assist her ventilations with a bag-mask device.

D

When caring for a woman who is experiencing a gynecologic emergency, the EMT’s main focus should be to:
A) Maintain her ABCs and transport without delay.

B) Keep assessment and treatment to a minimum.

C) Ask questions related to her gynecologic history.

D) Determine the underlying cause of her problem.

A

When caring for a female patient who has been sexually assaulted, you should:
A) Ask the patient for a concise, detailed report of what happened to her.

B) Place any bloodstained clothing or other articles in separate paper bags.

C) Advise her that she will not be allowed to shower or change her clothes.

D) Allow law enforcement to take her statement before you begin treatment.

B

Law enforcement personnel request your assistance to assess a 31-year-old female who was sexually assaulted. When you arrive at the scene, you find the patient sitting on a curb outside her apartment. She is conscious, alert, and crying. When you ask her what happened, she tells you that she does not want to be treated or transported to the hospital. She further tells you that all she wants to do is clean up. You should:
A) Ask her if there is anyone you can contact, such as a friend or relative.

B) Provide emotional support and visually assess her for obvious trauma.

C) Perform a limited hands-on assessment to detect life-threatening injuries.

D) Advise her that she cannot clean herself up because this will destroy evidence.

B

The physical examination of a sexual assault victim should be:
A) Limited to a brief survey for life-threatening injuries.

B) Performed in the presence of at least two police officers.

C) As detailed as possible so all injuries can be documented.

D) Deferred until the patient can be evaluated by a physician

A

Which of the following statements regarding the placenta is correct?
A) The placenta, also referred to as the afterbirth, provides oxygen and nutrients to the fetus and is expelled from the vagina about 30 minutes before the baby is born.

B) The placenta allows oxygen, carbon dioxide, and other products to transfer between the mother and fetus but does not allow blood to mix between the mother and fetus.

C) The placenta allows for the transfer of oxygen and carbon dioxide between the mother and fetus but prevents most medications from passing between the mother and fetus.

D) The placental barrier consists of two layers of cells and allows the mother’s blood that contains high concentrations of oxygen to directly mix with the blood of the fetus.

B

A mother who is pregnant with her first baby is typically in the first stage of labor for approximately:
A) 8 hours.

B) 10 hours.

C) 4 hours.

D) 16 hours.

D

After the fetus has descended into the pelvis at the end of the third trimester, many mothers experience:
A) A bloated feeling.

B) An urge to push.

C) Easier breathing.

D) Midback pain.

C

Signs and symptoms of preeclampsia include:
A) Dysuria and constipation.

B) Headache and edema.

C) Marked hypoglycemia.

D) Dyspnea and bradycardia.

B

Abruptio placenta occurs when:
A) The placenta affixes itself to the outer layer of the uterus.

B) A tear in the placenta causes severe internal hemorrhage.

C) The placenta prematurely separates from the uterine wall.

D) The placenta develops over and covers the cervical opening.

C

When preparing a pregnant patient for delivery, you should position her:
A) On a firm surface with her hips elevated 2 to 4.

B) In a supine position with her legs spread.

C) In a sitting position with her hips elevated 12.

D) On her left side with the right leg elevated.

A

When the mother is experiencing a contraction, you should instruct her to:
A) Take quick short breaths.

B) Rest and breathe deeply.

C) Hold her breath.

D) Push for 30 seconds.

A

During delivery of the baby’s head, you should suction the mouth before the nose because:
A) The mucosa of the nose is fragile and is easily damaged by vigorous suctioning.

B) It is easier to suction larger volumes of fluid from the baby’s oropharynx.

C) Suctioning the nose first may cause the baby to gasp and aspirate fluid.

D) Babies are primarily mouth breathers and do not breathe through their nose.

C

A newborn infant will usually begin breathing spontaneously within _______ seconds following birth.
A) 5 to 10

B) 30 to 60

C) 15 to 30

D) 3 to 5

C

The ONLY indications for placing your gloved fingers in the vagina during delivery are:
A) Vertex presentation and delivery of the placenta.

B) Breech presentation and prolapsed umbilical cord.

C) Limb presentation and severe vaginal hemorrhage.

D) Nuchal cord and presentation of an arm or leg.

B

Equipment and supplies that are carried on an ambulance should be stored:
A) In locked or secured cabinets in order to prevent theft.

B) Based on recommendations of the health department.

C) According to the urgency and frequency of their use.

D) As directed by the EMS system’s medical director.

C

Regardless of where portable and mounted oxygen cylinders are stored in the ambulance, they must:
A) Hold a minimum capacity of 1,500 L of oxygen.

B) Be capable of delivering oxygen at 1 to 15 L/min.

C) Be easily identifiable by their bright green color.

D) Undergo hydrostatic testing on a weekly basis.

B

Minimum staffing in the patient compartment of a basic life support (BLS) ambulance includes:
A) An EMT and a paramedic.

B) At least one EMT.

C) At least two EMTs.

D) An EMT and an AEMT.

B

Immediately upon arriving at the scene of an emergency call involving a traumatic injury, you should notify the dispatcher of your arrival and then:
A) Observe the scene for safety hazards.

B) Quickly gain access to the patient.

C) Determine if additional units are needed.

D) Carefully assess the mechanism of injury.

A

You have just delivered a major trauma patient to the hospital. Shortly after departing the hospital, dispatch advises you of another call. The back of the ambulance is contaminated with bloody dressings and is in disarray, and you are in need of airway equipment and numerous other supplies. You should:
A) Have your partner quickly clean the ambulance as you proceed to the call.

B) Proceed to the call, functioning only as an emergency medical responder.

C) Quickly proceed to the call and clean and restock the ambulance afterwards.

D) Advise the dispatcher that you are out of service and to send another unit.

D

After assessing your patient, you determine that his condition is stable. You provide the appropriate treatment and then load him into the ambulance. While en route to the hospital, you should:
A) Slow down and allow the driver to pass you.

B) Increase your speed to create more distance.

C) Slam on the brakes to frighten the tailgater.

D) Stop the ambulance and confront the driver.

A

The use of lights and siren on an ambulance:
A) Allows other drivers to hear and see you from a great distance.

B) Is required any time a patient is being transported to the hospital.

C) Signifies a request for other drivers to yield the right of way.

D) Legally gives the emergency vehicle operator the right of way.

C

Your unit and a fire department vehicle are responding to the scene of a patient in cardiac arrest. As you approach an intersection that is highly congested, you should:
A) Use a different siren tone than the fire department vehicle.

B) Turn off your lights and siren and proceed with safety.

C) Ask the fire department vehicle to remain 1,000 behind you.

D) Advise the fire department vehicle to follow you closely.

A

It is 10:30 PM and you have requested air medical transport for a critically injured patient. When you arrive at the designated landing zone, you should:
A) Mark the landing site using personnel with flashlights.

B) Survey the area for power lines or other hazards.

C) Mark the proposed landing area with road flares.

D) Provide the flight crew with a patient status update.

B

When being tailgated by another vehicle while responding to an emergency call, you should:
A) Slow down and allow the driver to pass you.

B) Increase your speed to create more distance.

C) Slam on the brakes to frighten the tailgater.

D) Stop the ambulance and confront the driver.

A

Extrication is MOST accurately defined as:
A) Using heavy equipment to access a patient.

B) Immobilizing a patient before moving him or her.

C) Removal from a dangerous situation or position.

D) Dismantling an automobile to remove a victim.

C

In order to evaluate hazards present at the scene and determine the number of patients, you should:
A) Interview bystanders present at the scene.

B) Request the fire department at all scenes.

C) Use the information provided by dispatch.

D) Perform a 360° walk-around of the scene.

D

You are attempting to gain access to a patient who was injured when his truck struck another vehicle from behind. The patient is conscious and alert, but is screaming in pain. You try to open the door, but it is locked. You should:
A) Use a pry bar to attempt to open the door.

B) Break the window and unlock the door.

C) Ask the patient if he can unlock the door.

D) Request the rescue team to extricate him.

C

A 50-year-old female is entrapped in her passenger car after it struck a tree. As the rescue team is preparing to extricate her, you quickly assess her and determine that she is breathing shallowly and that her radial pulse is absent. You should:
A) Begin CPR as the rescue team begins extrication.

B) Stabilize her condition before extrication begins.

C) Maintain spinal stabilization as she is extricated.

D) Secure her with a short backboard or vest device.

C

Disentanglement involves:
A) Removing a patient from a dangerous position.

B) Gaining access to a patient in a crashed vehicle.

C) Extrication techniques that EMTs are trained in.

D) The use of simple access tools such as a pry bar.

A

A hiker was injured when he fell approximately 20 from a cliff. When you arrive at the scene, a member of the technical rescue group escorts you to the patient, who is positioned on a steep incline. The MOST appropriate method of immobilizing and moving the patient to the ambulance is to:
A) Immobilize him with a short backboard and place him on the ambulance stretcher.

B) Immobilize him to a long backboard and use the four-person carry to move him.

C) Immobilize his spine with a long backboard and place him in a basket stretcher.

D) Apply a vest-style immobilization device and move him using a stair chair device.

C

Which of the following statements regarding trench rescue is correct?
A) Rescue vehicles should park at least 250 from the scene.

B) Ground vibration is a primary cause of secondary collapse.

C) Most deaths involving cave-ins are caused by head injury.

D) A trench deeper than 10 should be shored prior to entry.

B

You are standing by at the scene of a hostage situation when the incident commander advises you that one of his personnel has been shot. The patient is lying supine in an open area and is not moving. As the SWAT team escorts you to the patient, you should:
A) Treat only critical injuries before moving him to a safe place.

B) Grab him by the clothes and immediately move him to safety.

C) Perform a rapid assessment and move him to a place of safety.

D) Limit your primary assessment to airway and breathing only.

B

The reasons for rescue failure can be referred to by the mnemonic FAILURE. According to this mnemonic, the "U" stands for:
A) Unprepared to effectively manage the scene.

B) Undertrained to correctly utilize equipment.

C) Underutilizing personnel at the scene.

D) Underestimating the logistics of the incident.

D

You and your partner are standing by at the scene of a residential fire when you hear the incident commander state "We have located a victim" over the radio. You should:
A) Remain with the ambulance and wait for fire personnel to bring the victim to you.

B) Immediately locate the incident commander and ask where the victim is located.

C) Locate the victim and provide initial care while your partner stays with the ambulance.

D) Notify the hospital that you will be transporting a burn patient to their facility.

A

The function of the National Incident Management System (NIMS) is to:
A) Prepare for, prevent, respond to, and recover from domestic incidents.

B) Facilitate a standard method of incident command for natural disasters.

C) Educate city and county governments regarding foreign terrorist attacks.

D) Prepare for the potential of a nuclear attack against the United States.

A

Historically, the weak point at most major incidents has been:
A) Incident briefing.

B) Communication.

C) Accountability.

D) Lack of personnel.

B

When victims involved in a mass-casualty incident are moved to the treatment area:
A) All uninjured patients are placed in a holding area and closely observed.

B) Definitive care is provided and preparations for transport will be made.

C) They will be rapidly assessed and prioritized according to their injuries.

D) Secondary triage is performed and the appropriate treatment is rendered.

D

Burn patients without airway compromise and patients with multiple bone or joint injuries should be marked with a __________ triage at a mass-casualty incident.
A) Red

B) Green

C) Black

D) Yellow

D

The JumpSTART triage system is intended to be used for children younger than _____ years or who appear to weigh less than _____.
A) 8, 100 lb

B) 7, 90 lb

C) 6, 70 lb

D) 5, 50 lb

A

Unlike a mass-casualty incident, a natural disaster:
A) Is typically short-lived and does not require as much manpower.

B) Often requires personnel to remain on scene for several days.

C) Usually does not require the ICS process.

D) Exists when there are more than 100 critically injured patients.

B

A carboy is a container that would MOST likely be used to store and transport:
A) Combustible materials.

B) Corrosives.

C) Explosives.

D) Flammable liquids.

B

The MOST appropriate location to park your ambulance at a HazMat incident is:
A) Upwind at least 100 from the incident.

B) Downwind at least 200 from the scene.

C) Uphill at least 25 from the incident site.

D) Downhill at least 100 from the incident.

A

Which of the following activities occurs in the warm zone?
A) Command

B) Personnel staging

C) Medical monitoring

D) Decontamination

D

Following proper decontamination, a 30-year-old male is brought to you. He is semiconscious and has rapid, shallow respirations. A quick visual assessment reveals no obvious bleeding. You should:
A) Perform a rapid assessment to locate critical injuries.

B) Administer 100% oxygen via a nonrebreathing mask.

C) Ask a fire fighter what the patient was exposed to.

D) Begin some form of positive-pressure ventilation

D

The Centennial Park bombing during the 1996 Summer Olympics is an example of:
A) Domestic terrorism.

B) State-sponsored terrorism.

C) An ecoterrorist attack.

D) Apocalyptic violence.

A

A weapon of mass destruction (WMD) is MOST accurately defined as:
A) A nuclear or chemical weapon that can be launched from one country to another country.

B) Any agent used to bring about mass death, casualties, or massive infrastructural damage.

C) Any device used for the express purpose of creating carnage in an effort to make a particular point.

D) A device or agent used to destroy a specific area or region within a given geographic location.

B

Most terrorist attacks are:
A) Nuclear attacks.

B) Unplanned.

C) Covert.

D) Impulsive.

C

In determining the potential for a terrorist attack, you should routinely observe all of the following on every call, EXCEPT:
A) The location type.

B) Victim’s statements.

C) The type of call.

D) Weather conditions.

D

Cross-contamination occurs when:
A) An EMT provides care to a victim after the victim has been decontaminated.

B) An EMT has direct contact with a chemical agent at a terrorist incident.

C) An EMT is exposed to a victim who has not yet been decontaminated.

D) Two EMTs are exposed to the same agent after being decontaminated.

C

Which of the following agents blocks the body’s ability to use oxygen and possesses an odor similar to almonds?
A) Phosgene oxime

B) Carbon monoxide

C) Organophosphates

D) Hydrogen cyanide

D

The means by which a terrorist will spread a particular agent is called:
A) Dissemination.

B) Aerosolization.

C) Incubation.

D) Weaponization.

A

Early signs and symptoms of smallpox include all of the following, EXCEPT:
A) Skin blisters.

B) Body aches.

C) Headaches.

D) High fever.

A

When introduced into the body, ricin causes:
A) Intestinal obstruction and severe sepsis.

B) Necrosis of muscle tissue and cell destruction.

C) Pulmonary edema and circulatory failure.

D) Enlarged lymph nodes and extreme pain.

C

The LEAST harmful form of ionizing radiation is:
A) Alpha.

B) Beta.

C) Gamma.

D) Neutron.

A

The distal cuff on an ET tube should be inflated with no more than ______ of air after it is placed into the trachea.
A) 10 mL

B) 20 mL

C) 15 mL

D) 25 mL

A

You are transporting an intubated 24-year-old female when her level of consciousness improves and she becomes combative. Without assisted ventilation, she appears to be breathing adequately. You should anticipate that your ALS partner will:
A) Have suction available and remove the ET tube.

B) Extubate her to prevent laryngospasm.

C) Give her a sedative drug per local protocol.

D) Carefully restrain her and continue ventilations.

C

A major benefit when using a multilumen airway device is that:
A) Maintenance of a mask-to-face seal is not required.

B) It can be visualized as it enters the esophagus.

C) It can be used on patients of any age and size.

D) The airway is better protected than with an ET tube.

A

In the prehospital setting, gastric tubes are used primarily to:
A) Provide gastric nutrition.

B) Decompress the stomach.

C) Administer medications.

D) Remove gastric toxins.

B

For which of the following patients would the use of a continuous positive airway pressure (CPAP) device be of MOST benefit?
A) 60-year-old woman with emphysema, moderate shortness of breath, and a systolic blood pressure of 78 mm Hg

B) 58-year-old man with mild shortness of breath after being diagnosed by his physician with left lower lobe pneumonia

C) 62-year-old man with severe respiratory distress, a history of congestive heart failure, and audible rhonchi in the lungs

D) 50-year-old woman who is unable to follow verbal commands and is in severe respiratory distress due to asthma

C

After sliding the piercing spike of the administration set into the IV bag port, you should next:
A) Remove the protective cap and allow air to escape.

B) Prime the line and flush the air out of the tubing.

C) Ensure that the drip chamber is only half filled.

D) Adjust the drip chamber until fluid slowly drains.

B

If an IV line is found to be infiltrated, you should:
A) Immediately change the IV administration set.

B) Remove the catheter and apply direct pressure.

C) Slow the flow rate down to keep the vein open.

D) Apply a chemical heat pack to the IV site.

B

An IV line is not running as fast as it should be. There are no signs of infiltration. You should:
A) Manipulate the catheter until the flow improves.

B) Ensure that the tourniquet has been released.

C) Lower the IV bag as low as you possibly can.

D) Suspect that the patient has circulatory overload.

B

The QRS complex is an electrical representation of:
A) Atrial filling.

B) Atrial contraction.

C) Ventricular filling.

D) Ventricular contraction.

D

It does not matter if you place the arm leads on the patient’s shoulders or arms, as long as:
A) You apply the green lead first.

B) The red lead is placed on the right side.

C) They are at least 10 cm from the heart.

D) The patient is in a supine position.

C

Share This
Flashcard

More flashcards like this

NCLEX 10000 Integumentary Disorders

When assessing a client with partial-thickness burns over 60% of the body, which finding should the nurse report immediately? a) ...

Read more

NCLEX 300-NEURO

A client with amyotrophic lateral sclerosis (ALS) tells the nurse, "Sometimes I feel so frustrated. I can’t do anything without ...

Read more

NASM Flashcards

Which of the following is the process of getting oxygen from the environment to the tissues of the body? Diffusion ...

Read more

Unfinished tasks keep piling up?

Let us complete them for you. Quickly and professionally.

Check Price

Successful message
sending